Administración     

Olimpiadas de Matemáticas
Página de preparación y problemas

Selector
La base de datos contiene 1154 problemas y 775 soluciones.
OME Local
OME Nacional
OIM
OME Andalucía
Retos UJA
Problema 1027
Probar que hay infinitos números primos cuyo resto al dividirlos entre $3$ es $2$.
Sin pistas
Sin soluciones
info
Si crees que el enunciado contiene un error o imprecisión o bien crees que la información sobre la procedencia del problema es incorrecta, puedes notificarlo usando los siguientes botones:
Informar de error en enunciado Informar de procedencia del problema
Problema 1023
Describir todas las soluciones enteras positivas $(m,n)$ de la ecuación \[8m-7=n^2\] De entre todas las soluciones, calcular el menor valor de $m$ (si existe) mayor que $1959$.
Sin pistas
Sin soluciones
info
Si crees que el enunciado contiene un error o imprecisión o bien crees que la información sobre la procedencia del problema es incorrecta, puedes notificarlo usando los siguientes botones:
Informar de error en enunciado Informar de procedencia del problema
Problema 1019
Sean $k$ un entero positivo y $a_1,a_2,\ldots,a_k$ dígitos. Probar que existe un entero positivo $n$ tal que los últimos $2k$ dígitos de $2^n$ son, en este orden, $a_1,a_2,\ldots,a_k,b_1,b_2,\ldots,b_k$, para ciertos dígitos $b_1,b_2,\ldots,b_k$.
Sin pistas
Sin soluciones
info
Si crees que el enunciado contiene un error o imprecisión o bien crees que la información sobre la procedencia del problema es incorrecta, puedes notificarlo usando los siguientes botones:
Informar de error en enunciado Informar de procedencia del problema
Problema 1014
Determinar todos los números primos positivos $p,q,r,k$ tales que \[pq+qr+rp=12k+1.\]
pistasolución 1info
Pista. Trabaja módulo $4$ para demostrar que uno de los primos tiene que ser $2$. Luego trabaja módulo $3$.
Solución. Veamos en primer lugar que uno de los primos $p,q,r$ tiene que ser igual a $2$. Por reducción al absurdo, si los tres son impares, serán congruentes con $1$ o $3$ módulo $4$. Si los tres son congruentes con $1$ o los tres son congruentes con $3$, entonces $pq+qr+rp\equiv 1+1+1=3\ (\text{mod }4)$. También se tiene que si sólo uno o dos de ellos son congruentes con $1$, entonces $pq+qr+rp\equiv 1+3+3=3\ (\text{mod }4)$. Sin embargo, se tiene que $12k+1\equiv 1\ (\text{mod }4)$, lo cual es una contradicción.

Supongamos sin perder generalidad que $r=2$, luego la ecuación queda $pq+2p+2q=12k+1$. Vamos a probar ahora que uno de los primos $p,q$ es igual a $3$. De nuevo por reducción al absurdo, si $p$ y $q$ son congruentes con $1$ o con $2$ módulo $3$. Entonces, es fácil ver que $pq+2p+2q\equiv 0\ (\text{mod }3)$ si $p\equiv q\equiv 2$ o bien $pq+2p+2q\equiv 2\ (\text{mod }3)$ en caso contrario. No obstante, se tiene que $12k+1\equiv 1\ (\text{mod }3)$.

Podemos suponer entonces que $q=3$ sin perder generalidad y la ecuación original nos queda $5p+5=12k$. Como $k$ es primo y el miembro de la izquierda es múltiplo de $5$, tiene que ser $k=5$. Esto nos da $p=11$. Concluimos que las única posibilidad es que $p,q,r$ sean los primos $2,3,11$ (en cualquier orden) y $k=5$.

Si crees que el enunciado contiene un error o imprecisión o bien crees que la información sobre la procedencia del problema es incorrecta, puedes notificarlo usando los siguientes botones:
Informar de error en enunciado Informar de procedencia del problema
Problema 1011
Sea $m\geq 1$ un entero positivo y sean $a$ y $b$ enteros positivos distintos estrictamente comprendidos entre $m^2$ y $m^2+m$. Hallar todos los enteros $c$ estrictamente comprendidos entre $m^2$ y $m^2+m$ que dividen al producto $ab$.
Sin pistas
Sin soluciones
info
Si crees que el enunciado contiene un error o imprecisión o bien crees que la información sobre la procedencia del problema es incorrecta, puedes notificarlo usando los siguientes botones:
Informar de error en enunciado Informar de procedencia del problema
José Miguel Manzano © 2010-2024. Esta página ha sido creada mediante software libre